Why bikes hardly ever skid while braking with the front wheel?











up vote
21
down vote

favorite
2












When abruptly braking with the rear wheel on a bike, it tends to skid pretty easily. Doing the same with the front wheel is a very different experience. Instead of skidding, the bike lifts the rear wheel. I've never seen the front wheel of a bike skid.



According to this answer the torque generated on a bike during breaking is set around the front wheel contact point. While this explains why the rear wheel is lifted, it doesn't really explains why the wheel never slips.



What causes the front wheel to behave like it had more friction/grip with the ground?










share|cite|improve this question









New contributor




Tiago Marinho is a new contributor to this site. Take care in asking for clarification, commenting, and answering.
Check out our Code of Conduct.
















  • 1




    Do you mean drift (as in skid outwards on a bend "11. a controlled four-wheel skid, used by racing drivers to take bends at high speed " (Collins)) or simply skid in forward motion?
    – Chris H
    17 hours ago








  • 2




    I have managed to skid my front wheel (without crashing) but only in a dead-straight line, on wet leaves on tarmac. Any form of side-slipping the front wheel (while braking) is likely to be painful, and bikes never stay straight for long (there's another question in that). I've also skidded the front wheel on bends (with or without braking) and invariably hit the road rather hard.
    – Chris H
    17 hours ago












  • Skidding the front wheel occurs quite easily in deep, soft snow. It's also easier to handle than on wet road, because even though the wheel stops spinning it still works as a ski, retaining some control :)
    – jpa
    16 hours ago










  • This applies to all vehicles. In cars, since you can't control the front and rear brakes separately, it is designed such that the front brakes are applied with greater force than the rear brakes to ensure that all wheels lose adhesion and skid at roughly the same braking pressure. Engineers use the term brake bias to describe this.
    – user1936752
    14 hours ago












  • I've never seen the front wheel of a bike skid. grab a good fist full of front brake while you're riding on gravel, ice, wet metal or any other surface with diminished friction, and your story will abruptly change.. I recommend not to ride around utilizing the front brake as though it's impossible to lock the front wheel, if you wish to avoid injury..
    – Caius Jard
    9 hours ago

















up vote
21
down vote

favorite
2












When abruptly braking with the rear wheel on a bike, it tends to skid pretty easily. Doing the same with the front wheel is a very different experience. Instead of skidding, the bike lifts the rear wheel. I've never seen the front wheel of a bike skid.



According to this answer the torque generated on a bike during breaking is set around the front wheel contact point. While this explains why the rear wheel is lifted, it doesn't really explains why the wheel never slips.



What causes the front wheel to behave like it had more friction/grip with the ground?










share|cite|improve this question









New contributor




Tiago Marinho is a new contributor to this site. Take care in asking for clarification, commenting, and answering.
Check out our Code of Conduct.
















  • 1




    Do you mean drift (as in skid outwards on a bend "11. a controlled four-wheel skid, used by racing drivers to take bends at high speed " (Collins)) or simply skid in forward motion?
    – Chris H
    17 hours ago








  • 2




    I have managed to skid my front wheel (without crashing) but only in a dead-straight line, on wet leaves on tarmac. Any form of side-slipping the front wheel (while braking) is likely to be painful, and bikes never stay straight for long (there's another question in that). I've also skidded the front wheel on bends (with or without braking) and invariably hit the road rather hard.
    – Chris H
    17 hours ago












  • Skidding the front wheel occurs quite easily in deep, soft snow. It's also easier to handle than on wet road, because even though the wheel stops spinning it still works as a ski, retaining some control :)
    – jpa
    16 hours ago










  • This applies to all vehicles. In cars, since you can't control the front and rear brakes separately, it is designed such that the front brakes are applied with greater force than the rear brakes to ensure that all wheels lose adhesion and skid at roughly the same braking pressure. Engineers use the term brake bias to describe this.
    – user1936752
    14 hours ago












  • I've never seen the front wheel of a bike skid. grab a good fist full of front brake while you're riding on gravel, ice, wet metal or any other surface with diminished friction, and your story will abruptly change.. I recommend not to ride around utilizing the front brake as though it's impossible to lock the front wheel, if you wish to avoid injury..
    – Caius Jard
    9 hours ago















up vote
21
down vote

favorite
2









up vote
21
down vote

favorite
2






2





When abruptly braking with the rear wheel on a bike, it tends to skid pretty easily. Doing the same with the front wheel is a very different experience. Instead of skidding, the bike lifts the rear wheel. I've never seen the front wheel of a bike skid.



According to this answer the torque generated on a bike during breaking is set around the front wheel contact point. While this explains why the rear wheel is lifted, it doesn't really explains why the wheel never slips.



What causes the front wheel to behave like it had more friction/grip with the ground?










share|cite|improve this question









New contributor




Tiago Marinho is a new contributor to this site. Take care in asking for clarification, commenting, and answering.
Check out our Code of Conduct.











When abruptly braking with the rear wheel on a bike, it tends to skid pretty easily. Doing the same with the front wheel is a very different experience. Instead of skidding, the bike lifts the rear wheel. I've never seen the front wheel of a bike skid.



According to this answer the torque generated on a bike during breaking is set around the front wheel contact point. While this explains why the rear wheel is lifted, it doesn't really explains why the wheel never slips.



What causes the front wheel to behave like it had more friction/grip with the ground?







newtonian-mechanics rotational-dynamics friction everyday-life free-body-diagram






share|cite|improve this question









New contributor




Tiago Marinho is a new contributor to this site. Take care in asking for clarification, commenting, and answering.
Check out our Code of Conduct.











share|cite|improve this question









New contributor




Tiago Marinho is a new contributor to this site. Take care in asking for clarification, commenting, and answering.
Check out our Code of Conduct.









share|cite|improve this question




share|cite|improve this question








edited 11 hours ago









David Richerby

610815




610815






New contributor




Tiago Marinho is a new contributor to this site. Take care in asking for clarification, commenting, and answering.
Check out our Code of Conduct.









asked yesterday









Tiago Marinho

20914




20914




New contributor




Tiago Marinho is a new contributor to this site. Take care in asking for clarification, commenting, and answering.
Check out our Code of Conduct.





New contributor





Tiago Marinho is a new contributor to this site. Take care in asking for clarification, commenting, and answering.
Check out our Code of Conduct.






Tiago Marinho is a new contributor to this site. Take care in asking for clarification, commenting, and answering.
Check out our Code of Conduct.








  • 1




    Do you mean drift (as in skid outwards on a bend "11. a controlled four-wheel skid, used by racing drivers to take bends at high speed " (Collins)) or simply skid in forward motion?
    – Chris H
    17 hours ago








  • 2




    I have managed to skid my front wheel (without crashing) but only in a dead-straight line, on wet leaves on tarmac. Any form of side-slipping the front wheel (while braking) is likely to be painful, and bikes never stay straight for long (there's another question in that). I've also skidded the front wheel on bends (with or without braking) and invariably hit the road rather hard.
    – Chris H
    17 hours ago












  • Skidding the front wheel occurs quite easily in deep, soft snow. It's also easier to handle than on wet road, because even though the wheel stops spinning it still works as a ski, retaining some control :)
    – jpa
    16 hours ago










  • This applies to all vehicles. In cars, since you can't control the front and rear brakes separately, it is designed such that the front brakes are applied with greater force than the rear brakes to ensure that all wheels lose adhesion and skid at roughly the same braking pressure. Engineers use the term brake bias to describe this.
    – user1936752
    14 hours ago












  • I've never seen the front wheel of a bike skid. grab a good fist full of front brake while you're riding on gravel, ice, wet metal or any other surface with diminished friction, and your story will abruptly change.. I recommend not to ride around utilizing the front brake as though it's impossible to lock the front wheel, if you wish to avoid injury..
    – Caius Jard
    9 hours ago
















  • 1




    Do you mean drift (as in skid outwards on a bend "11. a controlled four-wheel skid, used by racing drivers to take bends at high speed " (Collins)) or simply skid in forward motion?
    – Chris H
    17 hours ago








  • 2




    I have managed to skid my front wheel (without crashing) but only in a dead-straight line, on wet leaves on tarmac. Any form of side-slipping the front wheel (while braking) is likely to be painful, and bikes never stay straight for long (there's another question in that). I've also skidded the front wheel on bends (with or without braking) and invariably hit the road rather hard.
    – Chris H
    17 hours ago












  • Skidding the front wheel occurs quite easily in deep, soft snow. It's also easier to handle than on wet road, because even though the wheel stops spinning it still works as a ski, retaining some control :)
    – jpa
    16 hours ago










  • This applies to all vehicles. In cars, since you can't control the front and rear brakes separately, it is designed such that the front brakes are applied with greater force than the rear brakes to ensure that all wheels lose adhesion and skid at roughly the same braking pressure. Engineers use the term brake bias to describe this.
    – user1936752
    14 hours ago












  • I've never seen the front wheel of a bike skid. grab a good fist full of front brake while you're riding on gravel, ice, wet metal or any other surface with diminished friction, and your story will abruptly change.. I recommend not to ride around utilizing the front brake as though it's impossible to lock the front wheel, if you wish to avoid injury..
    – Caius Jard
    9 hours ago










1




1




Do you mean drift (as in skid outwards on a bend "11. a controlled four-wheel skid, used by racing drivers to take bends at high speed " (Collins)) or simply skid in forward motion?
– Chris H
17 hours ago






Do you mean drift (as in skid outwards on a bend "11. a controlled four-wheel skid, used by racing drivers to take bends at high speed " (Collins)) or simply skid in forward motion?
– Chris H
17 hours ago






2




2




I have managed to skid my front wheel (without crashing) but only in a dead-straight line, on wet leaves on tarmac. Any form of side-slipping the front wheel (while braking) is likely to be painful, and bikes never stay straight for long (there's another question in that). I've also skidded the front wheel on bends (with or without braking) and invariably hit the road rather hard.
– Chris H
17 hours ago






I have managed to skid my front wheel (without crashing) but only in a dead-straight line, on wet leaves on tarmac. Any form of side-slipping the front wheel (while braking) is likely to be painful, and bikes never stay straight for long (there's another question in that). I've also skidded the front wheel on bends (with or without braking) and invariably hit the road rather hard.
– Chris H
17 hours ago














Skidding the front wheel occurs quite easily in deep, soft snow. It's also easier to handle than on wet road, because even though the wheel stops spinning it still works as a ski, retaining some control :)
– jpa
16 hours ago




Skidding the front wheel occurs quite easily in deep, soft snow. It's also easier to handle than on wet road, because even though the wheel stops spinning it still works as a ski, retaining some control :)
– jpa
16 hours ago












This applies to all vehicles. In cars, since you can't control the front and rear brakes separately, it is designed such that the front brakes are applied with greater force than the rear brakes to ensure that all wheels lose adhesion and skid at roughly the same braking pressure. Engineers use the term brake bias to describe this.
– user1936752
14 hours ago






This applies to all vehicles. In cars, since you can't control the front and rear brakes separately, it is designed such that the front brakes are applied with greater force than the rear brakes to ensure that all wheels lose adhesion and skid at roughly the same braking pressure. Engineers use the term brake bias to describe this.
– user1936752
14 hours ago














I've never seen the front wheel of a bike skid. grab a good fist full of front brake while you're riding on gravel, ice, wet metal or any other surface with diminished friction, and your story will abruptly change.. I recommend not to ride around utilizing the front brake as though it's impossible to lock the front wheel, if you wish to avoid injury..
– Caius Jard
9 hours ago






I've never seen the front wheel of a bike skid. grab a good fist full of front brake while you're riding on gravel, ice, wet metal or any other surface with diminished friction, and your story will abruptly change.. I recommend not to ride around utilizing the front brake as though it's impossible to lock the front wheel, if you wish to avoid injury..
– Caius Jard
9 hours ago












9 Answers
9






active

oldest

votes

















up vote
34
down vote













Forces shifting the weight to the front wheel when brakingBecause, as soon as you brake, all your weight is on the front wheel. The inertia coupled with gravity puts your weight and that of the bike onto the front wheel. More weight ⇒ more pressure ⇒ more friction/grip with the ground.






share|cite|improve this answer



















  • 2




    all your weight is on the front wheel Is it? The saddle is much closer to the rear wheel, almost above it.
    – henning
    20 hours ago






  • 3




    @henning But when you brake then the moment of inertia wants to rotate the bike (and the rider) such that there is more "weight" on the front wheel and less on the back.
    – phuzi
    20 hours ago






  • 10




    To be specific - if the rear wheel lifts then indeed all weight is on the front wheel, as explained by phuzi due to the forces affecting the bike. If you brake slightly lighter then some weight still remains on the rear wheel but still the force/weight on the front wheel is significantly larger.
    – Ister
    20 hours ago






  • 4




    and if you brake just a touch more than the back wheel lifting, you go over the bars and land on your head; maximal braking is close to instability. (@henning)
    – Chris H
    17 hours ago






  • 4




    @henning Incidentally, that's why you should almost never brake using the rear brake alone. As your weight shifts off the rear wheel while braking, the maximum frictional force of the rear wheel decreases. Using the front brake alone lets you stop in about half the distance as using the rear brake alone, and using both is even better.
    – Nuclear Wang
    16 hours ago


















up vote
9
down vote













In addition to Alex Doe's answer: front wheel drift does occur on slippery surfaces, Unfortunately a bike is really unstable in this mode so you'll fall over very soon (whereas a rear wheel drift is pretty stable and controllable).






share|cite|improve this answer








New contributor




Hobbes is a new contributor to this site. Take care in asking for clarification, commenting, and answering.
Check out our Code of Conduct.














  • 3




    Right. Correctly dosing an “anti-lock front brake maneuver” is an essentiall skill both for MTB and for survival in rainy cities without very well-designed cycle roads.
    – leftaroundabout
    18 hours ago










  • in a sharp turn, a front-wheel skid causes something that bikers call a high-side crash. they are spectacularly destructive and youtube is full of videos of them. search under "highside mulholland" to find some.
    – niels nielsen
    11 hours ago










  • CORRECTION! front-wheel drift causes a LOW-SIDE CRASH! My bad- NN
    – niels nielsen
    8 hours ago










  • @nielsnielsen I've managed to achieve both high and low-side crashes - depends if the tyre regains grip at any point after the loss of traction, and whether the weight is in a suitable place at that instant.
    – Criggie
    8 hours ago






  • 1




    A number of years ago now whilst on holiday in Corfu, I dropped a rental scooter in precisely this way. I braked too sharply with the front brake just as I hit a patch of gravel across the road. The front wheel locked, I couldn't control the skidding wheel, and everything went a bit pear-shaped. I was wearing jeans so my legs were only bruised, but I had a decent road rash on my left shoulder and arm.
    – Graham
    4 hours ago


















up vote
8
down vote













Why rear wheel easily drifts?



Consider the torque around contact of front tire from the frame of the cycle.




  • The pseudo force on the rider and the cycle is in forward direction


  • Thus, the weight and thus the normal force on rear tire tends to decrease.


  • Thus smaller friction force and bicycle tends to drift.



Why front wheel rarely drifts?




  • Considering the pseudo force and the tendency to rotate, the normal force on front tire is much larger (due to forward pseudo force on the centre of gravity). Thus, the grip is very tight.






share|cite|improve this answer




























    up vote
    5
    down vote














    it doesn't really explains why the wheel never slips




    Well, that's just not true. Front wheels can, and do slip. However, normally, the contact between front wheel and road is just too good for the slipping to set in before the wheel blocks and you go over the bars.



    It's actually all down to two factors:




    • the braking force that your front wheel can transmit to the road without slipping


    • the angle between the center of gravity, the wheel-road contact point, and the road



    The later factor is only a question of frame geometry. When your front wheel is about to block, the effective force on your center of gravity is exactly at that angle. Because gravity is fixed, and that angle is fixed, so is the braking force that is applied in this situation.



    Note that the entire weight of the bike and rider is on the front wheel when it's about to block, so the vertical force on the wheel-road contact point is fixed too.



    Now we can compare two forces:




    • The max braking force that your wheel material can transmit to the road in its current condition, given that the wheel is pressed to the road with the entire weight.


    • The braking force that is required to get you to the tipping point.



    If the first is the greater, your front wheel will block, and you will go over the bars. If the later is the greater, your front wheel will start slipping.





    Now, why is the first force the greater for almost all bikes?



    Well, a good tire allows about as much horizontal force to be transmitted to a dry road as the vertical force that is applied to press it onto the road. I.e., the angle between center of gravity, contact point, and road can get as low as 45°, and the front wheel is still able to block. Yet, most bikes are built in such a way that this angle is significantly greater than 45°.



    So, because of the geometry of most bikes, you need quite slippery road conditions in order for the front wheel to be able to slip. The longer the frame and the further back the center of gravity is behind the front wheel, the harder it becomes to block the front wheel and to go over the bars; up to the point that going over the bars becomes impossible.






    share|cite|improve this answer




























      up vote
      2
      down vote













      The brief answer to this is 'because the centre of mass of a bicycle is high up', and especially it is high up compared to a car, where often the front wheels can slip.



      To see how this works, consider a vastly oversimplified model of a bike: assume that the structure of the bike is light compared to its rider, and represent its rider as a point mass (see below for why this works even when that's not true). And we're going to be interested in the moment when the rear wheel lifts, so we can completely ignore the rear wheel, and concentrate just on the front wheel, and specifically on the point at which the front wheel touches the road. So the system looks something like this:



      enter image description here



      So, here, $c$ is the point at which the front tyre is touching the road, $m$ is the rider, and the horizontal & vertical distances between $c$ and $m$ are $l$ & $h$ respectively. And the bike is decelerating at $a$. And I've drawn the forces exerted by the front wheel on the road at $c$ (remember the back wheel is, by assumption, just lifting, so can be ignored: it's not exerting any forces on anything).



      For now assume that the coefficient of friction between the front wheel and the road is sufficiently high that the wheel does not slip and lets work out the point at which the bike just begins to tumble over the front wheel: this will tell us the maximum possible value of $a$, however sticky the front wheel is.



      It's pretty easy to see that the force on $m$ has two components: a vertical component which is $-mg$, where $g$ is acceleration due to gravity, and a horizontal component which is $ma$, where $a$ is the horizontal acceleration. And the bike will tumble when this vector points above the front wheel. Well, just by drawing the appropriate components you can see that this is true when



      $$frac{ma}{mg} gt frac{l}{h}$$



      or in other words, for the bike not to tumble



      $$a le frac{lg}{h}$$



      or



      $$a_text{max} = frac{lg}{h} tag{1}$$



      You can convince yourself this is right: a very tall bike ($h gg l$) will tumble really easily, and a completely flat bike ($h ll l$) will almost never tumble. And a bike in very low gravity will tumble more easily than one in high gravity. So (1) tells us how big $a$ can be, however sticky the front wheel is.



      Now consider the coefficient of friction at the front wheel. The coefficient of friction, $mu$ is defined as the force with which the wheel is trying to slide along the road and the force which it is being pressed down onto the road, at the point where the wheel just slips. So it's obvious that,



      $$mu = frac{ma_text{slip}}{mg} = frac{a_text{slip}}{g}$$



      where $a_text{slip}$ is the point at which the wheel slips. In other words



      $$a_text{slip} = mu g tag{2}$$



      And now we can use (1) & (2) to give us the answer we're looking for: the bike will tumble before it drifts if $a_text{slip} gt a_text{max}$, in other words if



      $$mu gt frac{l}{h}$$



      And you can now see the problem here: bikes are rather short and rather tall, so $l/h$ tends to be rather small, which means that the bike will tumble over the front wheel with a lower critical value of $mu$. And modern tyres on dry roads have values of $mu$ which can be fairly close to $1$ (I think $0.8$ to $0.9$ is plausible), while $l/h$ is generally significantly less than $1$.



      This is why bikes tumble before they drift.





      This approximation can be used even if the bike (or other vehicle) isn't light compared to the rider: you just need to work out where the centre of mass of the vehicle is and use that. For vehicles with suspension (some bikes have this of course, and even for bikes that don't the forks deflect under braking) you have to take into account the change in geometry under braking as well.






      share|cite|improve this answer






























        up vote
        0
        down vote













        I can't answer why but I can tell you that it is a property of multi-wheeled vehicles. Take a toy car and block the rear wheels so they don't move and push it on a smooth surface and you will see the locked rear wheels will always swap to the front. This is why a "bootleg turn" is possible by using the rear emergency brake to lock them down and the car will swap directions without changing the original vector by much. This is why the front brakes on a car does 90% of the braking and why the brakes on the front wheel of a motorcycle can help maintain control far more than the rear.






        share|cite|improve this answer








        New contributor




        Kathmandu Gilman is a new contributor to this site. Take care in asking for clarification, commenting, and answering.
        Check out our Code of Conduct.

























          up vote
          0
          down vote













          In the front wheel you have a much stronger vertical component when you use the breaks because of the momentum caused by inertia. With the distribution of forces and weight of a normal person, the back wheel will tend to lift adding more friction I'm the front wheel and stopping you harder and harder. To drift you need to have inertia in the back wheel but when you stop like this all the energy is released stopping you and lifting your back wheel



          When you use the break of the back wheel only, the momentum caused by inertia tends to lift the back wheel, such as in the case above. That causes friction to be lower I'm the back wheel and thus energy is not released suddenly, allowing you to drift.



          I believe that if you used a wheel with bad grip in the front wheel you should be able to drift






          share|cite|improve this answer








          New contributor




          F. S. is a new contributor to this site. Take care in asking for clarification, commenting, and answering.
          Check out our Code of Conduct.

























            up vote
            0
            down vote













            It's because the force of friction is approximately propertional to two things:




            • The "coefficient of friction" between the two surfaces (i.e. how sticky they are)

            • The "weight" or normal force (perpendicular force) between the two surfaces.


            The latter means, for example, that if you double the weight on a sled then you more or less double the friction between the sled and the ground.



            If you move all (or most of) your weight off the back wheel and onto the front wheel of a bike, then:




            • The force due to friction on the front wheel more or less doubles (i.e. you'd need twice as much sideways or backward force to make it skid).

            • The force due to friction on the back wheel goes towards zero (i.e. it skids more easily as it begins t lift off the ground)






            share|cite|improve this answer




























              up vote
              -3
              down vote













              I think its because you can turn the handle bars into the best angle to maintain your center of gravity near the body.



              While the back wheel cant turn so looses grip because the center of gravity goes outside your body during a drift






              share|cite|improve this answer




















                protected by ACuriousMind 11 hours ago



                Thank you for your interest in this question.
                Because it has attracted low-quality or spam answers that had to be removed, posting an answer now requires 10 reputation on this site (the association bonus does not count).



                Would you like to answer one of these unanswered questions instead?














                9 Answers
                9






                active

                oldest

                votes








                9 Answers
                9






                active

                oldest

                votes









                active

                oldest

                votes






                active

                oldest

                votes








                up vote
                34
                down vote













                Forces shifting the weight to the front wheel when brakingBecause, as soon as you brake, all your weight is on the front wheel. The inertia coupled with gravity puts your weight and that of the bike onto the front wheel. More weight ⇒ more pressure ⇒ more friction/grip with the ground.






                share|cite|improve this answer



















                • 2




                  all your weight is on the front wheel Is it? The saddle is much closer to the rear wheel, almost above it.
                  – henning
                  20 hours ago






                • 3




                  @henning But when you brake then the moment of inertia wants to rotate the bike (and the rider) such that there is more "weight" on the front wheel and less on the back.
                  – phuzi
                  20 hours ago






                • 10




                  To be specific - if the rear wheel lifts then indeed all weight is on the front wheel, as explained by phuzi due to the forces affecting the bike. If you brake slightly lighter then some weight still remains on the rear wheel but still the force/weight on the front wheel is significantly larger.
                  – Ister
                  20 hours ago






                • 4




                  and if you brake just a touch more than the back wheel lifting, you go over the bars and land on your head; maximal braking is close to instability. (@henning)
                  – Chris H
                  17 hours ago






                • 4




                  @henning Incidentally, that's why you should almost never brake using the rear brake alone. As your weight shifts off the rear wheel while braking, the maximum frictional force of the rear wheel decreases. Using the front brake alone lets you stop in about half the distance as using the rear brake alone, and using both is even better.
                  – Nuclear Wang
                  16 hours ago















                up vote
                34
                down vote













                Forces shifting the weight to the front wheel when brakingBecause, as soon as you brake, all your weight is on the front wheel. The inertia coupled with gravity puts your weight and that of the bike onto the front wheel. More weight ⇒ more pressure ⇒ more friction/grip with the ground.






                share|cite|improve this answer



















                • 2




                  all your weight is on the front wheel Is it? The saddle is much closer to the rear wheel, almost above it.
                  – henning
                  20 hours ago






                • 3




                  @henning But when you brake then the moment of inertia wants to rotate the bike (and the rider) such that there is more "weight" on the front wheel and less on the back.
                  – phuzi
                  20 hours ago






                • 10




                  To be specific - if the rear wheel lifts then indeed all weight is on the front wheel, as explained by phuzi due to the forces affecting the bike. If you brake slightly lighter then some weight still remains on the rear wheel but still the force/weight on the front wheel is significantly larger.
                  – Ister
                  20 hours ago






                • 4




                  and if you brake just a touch more than the back wheel lifting, you go over the bars and land on your head; maximal braking is close to instability. (@henning)
                  – Chris H
                  17 hours ago






                • 4




                  @henning Incidentally, that's why you should almost never brake using the rear brake alone. As your weight shifts off the rear wheel while braking, the maximum frictional force of the rear wheel decreases. Using the front brake alone lets you stop in about half the distance as using the rear brake alone, and using both is even better.
                  – Nuclear Wang
                  16 hours ago













                up vote
                34
                down vote










                up vote
                34
                down vote









                Forces shifting the weight to the front wheel when brakingBecause, as soon as you brake, all your weight is on the front wheel. The inertia coupled with gravity puts your weight and that of the bike onto the front wheel. More weight ⇒ more pressure ⇒ more friction/grip with the ground.






                share|cite|improve this answer














                Forces shifting the weight to the front wheel when brakingBecause, as soon as you brake, all your weight is on the front wheel. The inertia coupled with gravity puts your weight and that of the bike onto the front wheel. More weight ⇒ more pressure ⇒ more friction/grip with the ground.







                share|cite|improve this answer














                share|cite|improve this answer



                share|cite|improve this answer








                edited 17 hours ago

























                answered yesterday









                Alex Doe

                37119




                37119








                • 2




                  all your weight is on the front wheel Is it? The saddle is much closer to the rear wheel, almost above it.
                  – henning
                  20 hours ago






                • 3




                  @henning But when you brake then the moment of inertia wants to rotate the bike (and the rider) such that there is more "weight" on the front wheel and less on the back.
                  – phuzi
                  20 hours ago






                • 10




                  To be specific - if the rear wheel lifts then indeed all weight is on the front wheel, as explained by phuzi due to the forces affecting the bike. If you brake slightly lighter then some weight still remains on the rear wheel but still the force/weight on the front wheel is significantly larger.
                  – Ister
                  20 hours ago






                • 4




                  and if you brake just a touch more than the back wheel lifting, you go over the bars and land on your head; maximal braking is close to instability. (@henning)
                  – Chris H
                  17 hours ago






                • 4




                  @henning Incidentally, that's why you should almost never brake using the rear brake alone. As your weight shifts off the rear wheel while braking, the maximum frictional force of the rear wheel decreases. Using the front brake alone lets you stop in about half the distance as using the rear brake alone, and using both is even better.
                  – Nuclear Wang
                  16 hours ago














                • 2




                  all your weight is on the front wheel Is it? The saddle is much closer to the rear wheel, almost above it.
                  – henning
                  20 hours ago






                • 3




                  @henning But when you brake then the moment of inertia wants to rotate the bike (and the rider) such that there is more "weight" on the front wheel and less on the back.
                  – phuzi
                  20 hours ago






                • 10




                  To be specific - if the rear wheel lifts then indeed all weight is on the front wheel, as explained by phuzi due to the forces affecting the bike. If you brake slightly lighter then some weight still remains on the rear wheel but still the force/weight on the front wheel is significantly larger.
                  – Ister
                  20 hours ago






                • 4




                  and if you brake just a touch more than the back wheel lifting, you go over the bars and land on your head; maximal braking is close to instability. (@henning)
                  – Chris H
                  17 hours ago






                • 4




                  @henning Incidentally, that's why you should almost never brake using the rear brake alone. As your weight shifts off the rear wheel while braking, the maximum frictional force of the rear wheel decreases. Using the front brake alone lets you stop in about half the distance as using the rear brake alone, and using both is even better.
                  – Nuclear Wang
                  16 hours ago








                2




                2




                all your weight is on the front wheel Is it? The saddle is much closer to the rear wheel, almost above it.
                – henning
                20 hours ago




                all your weight is on the front wheel Is it? The saddle is much closer to the rear wheel, almost above it.
                – henning
                20 hours ago




                3




                3




                @henning But when you brake then the moment of inertia wants to rotate the bike (and the rider) such that there is more "weight" on the front wheel and less on the back.
                – phuzi
                20 hours ago




                @henning But when you brake then the moment of inertia wants to rotate the bike (and the rider) such that there is more "weight" on the front wheel and less on the back.
                – phuzi
                20 hours ago




                10




                10




                To be specific - if the rear wheel lifts then indeed all weight is on the front wheel, as explained by phuzi due to the forces affecting the bike. If you brake slightly lighter then some weight still remains on the rear wheel but still the force/weight on the front wheel is significantly larger.
                – Ister
                20 hours ago




                To be specific - if the rear wheel lifts then indeed all weight is on the front wheel, as explained by phuzi due to the forces affecting the bike. If you brake slightly lighter then some weight still remains on the rear wheel but still the force/weight on the front wheel is significantly larger.
                – Ister
                20 hours ago




                4




                4




                and if you brake just a touch more than the back wheel lifting, you go over the bars and land on your head; maximal braking is close to instability. (@henning)
                – Chris H
                17 hours ago




                and if you brake just a touch more than the back wheel lifting, you go over the bars and land on your head; maximal braking is close to instability. (@henning)
                – Chris H
                17 hours ago




                4




                4




                @henning Incidentally, that's why you should almost never brake using the rear brake alone. As your weight shifts off the rear wheel while braking, the maximum frictional force of the rear wheel decreases. Using the front brake alone lets you stop in about half the distance as using the rear brake alone, and using both is even better.
                – Nuclear Wang
                16 hours ago




                @henning Incidentally, that's why you should almost never brake using the rear brake alone. As your weight shifts off the rear wheel while braking, the maximum frictional force of the rear wheel decreases. Using the front brake alone lets you stop in about half the distance as using the rear brake alone, and using both is even better.
                – Nuclear Wang
                16 hours ago










                up vote
                9
                down vote













                In addition to Alex Doe's answer: front wheel drift does occur on slippery surfaces, Unfortunately a bike is really unstable in this mode so you'll fall over very soon (whereas a rear wheel drift is pretty stable and controllable).






                share|cite|improve this answer








                New contributor




                Hobbes is a new contributor to this site. Take care in asking for clarification, commenting, and answering.
                Check out our Code of Conduct.














                • 3




                  Right. Correctly dosing an “anti-lock front brake maneuver” is an essentiall skill both for MTB and for survival in rainy cities without very well-designed cycle roads.
                  – leftaroundabout
                  18 hours ago










                • in a sharp turn, a front-wheel skid causes something that bikers call a high-side crash. they are spectacularly destructive and youtube is full of videos of them. search under "highside mulholland" to find some.
                  – niels nielsen
                  11 hours ago










                • CORRECTION! front-wheel drift causes a LOW-SIDE CRASH! My bad- NN
                  – niels nielsen
                  8 hours ago










                • @nielsnielsen I've managed to achieve both high and low-side crashes - depends if the tyre regains grip at any point after the loss of traction, and whether the weight is in a suitable place at that instant.
                  – Criggie
                  8 hours ago






                • 1




                  A number of years ago now whilst on holiday in Corfu, I dropped a rental scooter in precisely this way. I braked too sharply with the front brake just as I hit a patch of gravel across the road. The front wheel locked, I couldn't control the skidding wheel, and everything went a bit pear-shaped. I was wearing jeans so my legs were only bruised, but I had a decent road rash on my left shoulder and arm.
                  – Graham
                  4 hours ago















                up vote
                9
                down vote













                In addition to Alex Doe's answer: front wheel drift does occur on slippery surfaces, Unfortunately a bike is really unstable in this mode so you'll fall over very soon (whereas a rear wheel drift is pretty stable and controllable).






                share|cite|improve this answer








                New contributor




                Hobbes is a new contributor to this site. Take care in asking for clarification, commenting, and answering.
                Check out our Code of Conduct.














                • 3




                  Right. Correctly dosing an “anti-lock front brake maneuver” is an essentiall skill both for MTB and for survival in rainy cities without very well-designed cycle roads.
                  – leftaroundabout
                  18 hours ago










                • in a sharp turn, a front-wheel skid causes something that bikers call a high-side crash. they are spectacularly destructive and youtube is full of videos of them. search under "highside mulholland" to find some.
                  – niels nielsen
                  11 hours ago










                • CORRECTION! front-wheel drift causes a LOW-SIDE CRASH! My bad- NN
                  – niels nielsen
                  8 hours ago










                • @nielsnielsen I've managed to achieve both high and low-side crashes - depends if the tyre regains grip at any point after the loss of traction, and whether the weight is in a suitable place at that instant.
                  – Criggie
                  8 hours ago






                • 1




                  A number of years ago now whilst on holiday in Corfu, I dropped a rental scooter in precisely this way. I braked too sharply with the front brake just as I hit a patch of gravel across the road. The front wheel locked, I couldn't control the skidding wheel, and everything went a bit pear-shaped. I was wearing jeans so my legs were only bruised, but I had a decent road rash on my left shoulder and arm.
                  – Graham
                  4 hours ago













                up vote
                9
                down vote










                up vote
                9
                down vote









                In addition to Alex Doe's answer: front wheel drift does occur on slippery surfaces, Unfortunately a bike is really unstable in this mode so you'll fall over very soon (whereas a rear wheel drift is pretty stable and controllable).






                share|cite|improve this answer








                New contributor




                Hobbes is a new contributor to this site. Take care in asking for clarification, commenting, and answering.
                Check out our Code of Conduct.









                In addition to Alex Doe's answer: front wheel drift does occur on slippery surfaces, Unfortunately a bike is really unstable in this mode so you'll fall over very soon (whereas a rear wheel drift is pretty stable and controllable).







                share|cite|improve this answer








                New contributor




                Hobbes is a new contributor to this site. Take care in asking for clarification, commenting, and answering.
                Check out our Code of Conduct.









                share|cite|improve this answer



                share|cite|improve this answer






                New contributor




                Hobbes is a new contributor to this site. Take care in asking for clarification, commenting, and answering.
                Check out our Code of Conduct.









                answered 21 hours ago









                Hobbes

                1912




                1912




                New contributor




                Hobbes is a new contributor to this site. Take care in asking for clarification, commenting, and answering.
                Check out our Code of Conduct.





                New contributor





                Hobbes is a new contributor to this site. Take care in asking for clarification, commenting, and answering.
                Check out our Code of Conduct.






                Hobbes is a new contributor to this site. Take care in asking for clarification, commenting, and answering.
                Check out our Code of Conduct.








                • 3




                  Right. Correctly dosing an “anti-lock front brake maneuver” is an essentiall skill both for MTB and for survival in rainy cities without very well-designed cycle roads.
                  – leftaroundabout
                  18 hours ago










                • in a sharp turn, a front-wheel skid causes something that bikers call a high-side crash. they are spectacularly destructive and youtube is full of videos of them. search under "highside mulholland" to find some.
                  – niels nielsen
                  11 hours ago










                • CORRECTION! front-wheel drift causes a LOW-SIDE CRASH! My bad- NN
                  – niels nielsen
                  8 hours ago










                • @nielsnielsen I've managed to achieve both high and low-side crashes - depends if the tyre regains grip at any point after the loss of traction, and whether the weight is in a suitable place at that instant.
                  – Criggie
                  8 hours ago






                • 1




                  A number of years ago now whilst on holiday in Corfu, I dropped a rental scooter in precisely this way. I braked too sharply with the front brake just as I hit a patch of gravel across the road. The front wheel locked, I couldn't control the skidding wheel, and everything went a bit pear-shaped. I was wearing jeans so my legs were only bruised, but I had a decent road rash on my left shoulder and arm.
                  – Graham
                  4 hours ago














                • 3




                  Right. Correctly dosing an “anti-lock front brake maneuver” is an essentiall skill both for MTB and for survival in rainy cities without very well-designed cycle roads.
                  – leftaroundabout
                  18 hours ago










                • in a sharp turn, a front-wheel skid causes something that bikers call a high-side crash. they are spectacularly destructive and youtube is full of videos of them. search under "highside mulholland" to find some.
                  – niels nielsen
                  11 hours ago










                • CORRECTION! front-wheel drift causes a LOW-SIDE CRASH! My bad- NN
                  – niels nielsen
                  8 hours ago










                • @nielsnielsen I've managed to achieve both high and low-side crashes - depends if the tyre regains grip at any point after the loss of traction, and whether the weight is in a suitable place at that instant.
                  – Criggie
                  8 hours ago






                • 1




                  A number of years ago now whilst on holiday in Corfu, I dropped a rental scooter in precisely this way. I braked too sharply with the front brake just as I hit a patch of gravel across the road. The front wheel locked, I couldn't control the skidding wheel, and everything went a bit pear-shaped. I was wearing jeans so my legs were only bruised, but I had a decent road rash on my left shoulder and arm.
                  – Graham
                  4 hours ago








                3




                3




                Right. Correctly dosing an “anti-lock front brake maneuver” is an essentiall skill both for MTB and for survival in rainy cities without very well-designed cycle roads.
                – leftaroundabout
                18 hours ago




                Right. Correctly dosing an “anti-lock front brake maneuver” is an essentiall skill both for MTB and for survival in rainy cities without very well-designed cycle roads.
                – leftaroundabout
                18 hours ago












                in a sharp turn, a front-wheel skid causes something that bikers call a high-side crash. they are spectacularly destructive and youtube is full of videos of them. search under "highside mulholland" to find some.
                – niels nielsen
                11 hours ago




                in a sharp turn, a front-wheel skid causes something that bikers call a high-side crash. they are spectacularly destructive and youtube is full of videos of them. search under "highside mulholland" to find some.
                – niels nielsen
                11 hours ago












                CORRECTION! front-wheel drift causes a LOW-SIDE CRASH! My bad- NN
                – niels nielsen
                8 hours ago




                CORRECTION! front-wheel drift causes a LOW-SIDE CRASH! My bad- NN
                – niels nielsen
                8 hours ago












                @nielsnielsen I've managed to achieve both high and low-side crashes - depends if the tyre regains grip at any point after the loss of traction, and whether the weight is in a suitable place at that instant.
                – Criggie
                8 hours ago




                @nielsnielsen I've managed to achieve both high and low-side crashes - depends if the tyre regains grip at any point after the loss of traction, and whether the weight is in a suitable place at that instant.
                – Criggie
                8 hours ago




                1




                1




                A number of years ago now whilst on holiday in Corfu, I dropped a rental scooter in precisely this way. I braked too sharply with the front brake just as I hit a patch of gravel across the road. The front wheel locked, I couldn't control the skidding wheel, and everything went a bit pear-shaped. I was wearing jeans so my legs were only bruised, but I had a decent road rash on my left shoulder and arm.
                – Graham
                4 hours ago




                A number of years ago now whilst on holiday in Corfu, I dropped a rental scooter in precisely this way. I braked too sharply with the front brake just as I hit a patch of gravel across the road. The front wheel locked, I couldn't control the skidding wheel, and everything went a bit pear-shaped. I was wearing jeans so my legs were only bruised, but I had a decent road rash on my left shoulder and arm.
                – Graham
                4 hours ago










                up vote
                8
                down vote













                Why rear wheel easily drifts?



                Consider the torque around contact of front tire from the frame of the cycle.




                • The pseudo force on the rider and the cycle is in forward direction


                • Thus, the weight and thus the normal force on rear tire tends to decrease.


                • Thus smaller friction force and bicycle tends to drift.



                Why front wheel rarely drifts?




                • Considering the pseudo force and the tendency to rotate, the normal force on front tire is much larger (due to forward pseudo force on the centre of gravity). Thus, the grip is very tight.






                share|cite|improve this answer

























                  up vote
                  8
                  down vote













                  Why rear wheel easily drifts?



                  Consider the torque around contact of front tire from the frame of the cycle.




                  • The pseudo force on the rider and the cycle is in forward direction


                  • Thus, the weight and thus the normal force on rear tire tends to decrease.


                  • Thus smaller friction force and bicycle tends to drift.



                  Why front wheel rarely drifts?




                  • Considering the pseudo force and the tendency to rotate, the normal force on front tire is much larger (due to forward pseudo force on the centre of gravity). Thus, the grip is very tight.






                  share|cite|improve this answer























                    up vote
                    8
                    down vote










                    up vote
                    8
                    down vote









                    Why rear wheel easily drifts?



                    Consider the torque around contact of front tire from the frame of the cycle.




                    • The pseudo force on the rider and the cycle is in forward direction


                    • Thus, the weight and thus the normal force on rear tire tends to decrease.


                    • Thus smaller friction force and bicycle tends to drift.



                    Why front wheel rarely drifts?




                    • Considering the pseudo force and the tendency to rotate, the normal force on front tire is much larger (due to forward pseudo force on the centre of gravity). Thus, the grip is very tight.






                    share|cite|improve this answer












                    Why rear wheel easily drifts?



                    Consider the torque around contact of front tire from the frame of the cycle.




                    • The pseudo force on the rider and the cycle is in forward direction


                    • Thus, the weight and thus the normal force on rear tire tends to decrease.


                    • Thus smaller friction force and bicycle tends to drift.



                    Why front wheel rarely drifts?




                    • Considering the pseudo force and the tendency to rotate, the normal force on front tire is much larger (due to forward pseudo force on the centre of gravity). Thus, the grip is very tight.







                    share|cite|improve this answer












                    share|cite|improve this answer



                    share|cite|improve this answer










                    answered yesterday









                    m__

                    1479




                    1479






















                        up vote
                        5
                        down vote














                        it doesn't really explains why the wheel never slips




                        Well, that's just not true. Front wheels can, and do slip. However, normally, the contact between front wheel and road is just too good for the slipping to set in before the wheel blocks and you go over the bars.



                        It's actually all down to two factors:




                        • the braking force that your front wheel can transmit to the road without slipping


                        • the angle between the center of gravity, the wheel-road contact point, and the road



                        The later factor is only a question of frame geometry. When your front wheel is about to block, the effective force on your center of gravity is exactly at that angle. Because gravity is fixed, and that angle is fixed, so is the braking force that is applied in this situation.



                        Note that the entire weight of the bike and rider is on the front wheel when it's about to block, so the vertical force on the wheel-road contact point is fixed too.



                        Now we can compare two forces:




                        • The max braking force that your wheel material can transmit to the road in its current condition, given that the wheel is pressed to the road with the entire weight.


                        • The braking force that is required to get you to the tipping point.



                        If the first is the greater, your front wheel will block, and you will go over the bars. If the later is the greater, your front wheel will start slipping.





                        Now, why is the first force the greater for almost all bikes?



                        Well, a good tire allows about as much horizontal force to be transmitted to a dry road as the vertical force that is applied to press it onto the road. I.e., the angle between center of gravity, contact point, and road can get as low as 45°, and the front wheel is still able to block. Yet, most bikes are built in such a way that this angle is significantly greater than 45°.



                        So, because of the geometry of most bikes, you need quite slippery road conditions in order for the front wheel to be able to slip. The longer the frame and the further back the center of gravity is behind the front wheel, the harder it becomes to block the front wheel and to go over the bars; up to the point that going over the bars becomes impossible.






                        share|cite|improve this answer

























                          up vote
                          5
                          down vote














                          it doesn't really explains why the wheel never slips




                          Well, that's just not true. Front wheels can, and do slip. However, normally, the contact between front wheel and road is just too good for the slipping to set in before the wheel blocks and you go over the bars.



                          It's actually all down to two factors:




                          • the braking force that your front wheel can transmit to the road without slipping


                          • the angle between the center of gravity, the wheel-road contact point, and the road



                          The later factor is only a question of frame geometry. When your front wheel is about to block, the effective force on your center of gravity is exactly at that angle. Because gravity is fixed, and that angle is fixed, so is the braking force that is applied in this situation.



                          Note that the entire weight of the bike and rider is on the front wheel when it's about to block, so the vertical force on the wheel-road contact point is fixed too.



                          Now we can compare two forces:




                          • The max braking force that your wheel material can transmit to the road in its current condition, given that the wheel is pressed to the road with the entire weight.


                          • The braking force that is required to get you to the tipping point.



                          If the first is the greater, your front wheel will block, and you will go over the bars. If the later is the greater, your front wheel will start slipping.





                          Now, why is the first force the greater for almost all bikes?



                          Well, a good tire allows about as much horizontal force to be transmitted to a dry road as the vertical force that is applied to press it onto the road. I.e., the angle between center of gravity, contact point, and road can get as low as 45°, and the front wheel is still able to block. Yet, most bikes are built in such a way that this angle is significantly greater than 45°.



                          So, because of the geometry of most bikes, you need quite slippery road conditions in order for the front wheel to be able to slip. The longer the frame and the further back the center of gravity is behind the front wheel, the harder it becomes to block the front wheel and to go over the bars; up to the point that going over the bars becomes impossible.






                          share|cite|improve this answer























                            up vote
                            5
                            down vote










                            up vote
                            5
                            down vote










                            it doesn't really explains why the wheel never slips




                            Well, that's just not true. Front wheels can, and do slip. However, normally, the contact between front wheel and road is just too good for the slipping to set in before the wheel blocks and you go over the bars.



                            It's actually all down to two factors:




                            • the braking force that your front wheel can transmit to the road without slipping


                            • the angle between the center of gravity, the wheel-road contact point, and the road



                            The later factor is only a question of frame geometry. When your front wheel is about to block, the effective force on your center of gravity is exactly at that angle. Because gravity is fixed, and that angle is fixed, so is the braking force that is applied in this situation.



                            Note that the entire weight of the bike and rider is on the front wheel when it's about to block, so the vertical force on the wheel-road contact point is fixed too.



                            Now we can compare two forces:




                            • The max braking force that your wheel material can transmit to the road in its current condition, given that the wheel is pressed to the road with the entire weight.


                            • The braking force that is required to get you to the tipping point.



                            If the first is the greater, your front wheel will block, and you will go over the bars. If the later is the greater, your front wheel will start slipping.





                            Now, why is the first force the greater for almost all bikes?



                            Well, a good tire allows about as much horizontal force to be transmitted to a dry road as the vertical force that is applied to press it onto the road. I.e., the angle between center of gravity, contact point, and road can get as low as 45°, and the front wheel is still able to block. Yet, most bikes are built in such a way that this angle is significantly greater than 45°.



                            So, because of the geometry of most bikes, you need quite slippery road conditions in order for the front wheel to be able to slip. The longer the frame and the further back the center of gravity is behind the front wheel, the harder it becomes to block the front wheel and to go over the bars; up to the point that going over the bars becomes impossible.






                            share|cite|improve this answer













                            it doesn't really explains why the wheel never slips




                            Well, that's just not true. Front wheels can, and do slip. However, normally, the contact between front wheel and road is just too good for the slipping to set in before the wheel blocks and you go over the bars.



                            It's actually all down to two factors:




                            • the braking force that your front wheel can transmit to the road without slipping


                            • the angle between the center of gravity, the wheel-road contact point, and the road



                            The later factor is only a question of frame geometry. When your front wheel is about to block, the effective force on your center of gravity is exactly at that angle. Because gravity is fixed, and that angle is fixed, so is the braking force that is applied in this situation.



                            Note that the entire weight of the bike and rider is on the front wheel when it's about to block, so the vertical force on the wheel-road contact point is fixed too.



                            Now we can compare two forces:




                            • The max braking force that your wheel material can transmit to the road in its current condition, given that the wheel is pressed to the road with the entire weight.


                            • The braking force that is required to get you to the tipping point.



                            If the first is the greater, your front wheel will block, and you will go over the bars. If the later is the greater, your front wheel will start slipping.





                            Now, why is the first force the greater for almost all bikes?



                            Well, a good tire allows about as much horizontal force to be transmitted to a dry road as the vertical force that is applied to press it onto the road. I.e., the angle between center of gravity, contact point, and road can get as low as 45°, and the front wheel is still able to block. Yet, most bikes are built in such a way that this angle is significantly greater than 45°.



                            So, because of the geometry of most bikes, you need quite slippery road conditions in order for the front wheel to be able to slip. The longer the frame and the further back the center of gravity is behind the front wheel, the harder it becomes to block the front wheel and to go over the bars; up to the point that going over the bars becomes impossible.







                            share|cite|improve this answer












                            share|cite|improve this answer



                            share|cite|improve this answer










                            answered 19 hours ago









                            cmaster

                            4292




                            4292






















                                up vote
                                2
                                down vote













                                The brief answer to this is 'because the centre of mass of a bicycle is high up', and especially it is high up compared to a car, where often the front wheels can slip.



                                To see how this works, consider a vastly oversimplified model of a bike: assume that the structure of the bike is light compared to its rider, and represent its rider as a point mass (see below for why this works even when that's not true). And we're going to be interested in the moment when the rear wheel lifts, so we can completely ignore the rear wheel, and concentrate just on the front wheel, and specifically on the point at which the front wheel touches the road. So the system looks something like this:



                                enter image description here



                                So, here, $c$ is the point at which the front tyre is touching the road, $m$ is the rider, and the horizontal & vertical distances between $c$ and $m$ are $l$ & $h$ respectively. And the bike is decelerating at $a$. And I've drawn the forces exerted by the front wheel on the road at $c$ (remember the back wheel is, by assumption, just lifting, so can be ignored: it's not exerting any forces on anything).



                                For now assume that the coefficient of friction between the front wheel and the road is sufficiently high that the wheel does not slip and lets work out the point at which the bike just begins to tumble over the front wheel: this will tell us the maximum possible value of $a$, however sticky the front wheel is.



                                It's pretty easy to see that the force on $m$ has two components: a vertical component which is $-mg$, where $g$ is acceleration due to gravity, and a horizontal component which is $ma$, where $a$ is the horizontal acceleration. And the bike will tumble when this vector points above the front wheel. Well, just by drawing the appropriate components you can see that this is true when



                                $$frac{ma}{mg} gt frac{l}{h}$$



                                or in other words, for the bike not to tumble



                                $$a le frac{lg}{h}$$



                                or



                                $$a_text{max} = frac{lg}{h} tag{1}$$



                                You can convince yourself this is right: a very tall bike ($h gg l$) will tumble really easily, and a completely flat bike ($h ll l$) will almost never tumble. And a bike in very low gravity will tumble more easily than one in high gravity. So (1) tells us how big $a$ can be, however sticky the front wheel is.



                                Now consider the coefficient of friction at the front wheel. The coefficient of friction, $mu$ is defined as the force with which the wheel is trying to slide along the road and the force which it is being pressed down onto the road, at the point where the wheel just slips. So it's obvious that,



                                $$mu = frac{ma_text{slip}}{mg} = frac{a_text{slip}}{g}$$



                                where $a_text{slip}$ is the point at which the wheel slips. In other words



                                $$a_text{slip} = mu g tag{2}$$



                                And now we can use (1) & (2) to give us the answer we're looking for: the bike will tumble before it drifts if $a_text{slip} gt a_text{max}$, in other words if



                                $$mu gt frac{l}{h}$$



                                And you can now see the problem here: bikes are rather short and rather tall, so $l/h$ tends to be rather small, which means that the bike will tumble over the front wheel with a lower critical value of $mu$. And modern tyres on dry roads have values of $mu$ which can be fairly close to $1$ (I think $0.8$ to $0.9$ is plausible), while $l/h$ is generally significantly less than $1$.



                                This is why bikes tumble before they drift.





                                This approximation can be used even if the bike (or other vehicle) isn't light compared to the rider: you just need to work out where the centre of mass of the vehicle is and use that. For vehicles with suspension (some bikes have this of course, and even for bikes that don't the forks deflect under braking) you have to take into account the change in geometry under braking as well.






                                share|cite|improve this answer



























                                  up vote
                                  2
                                  down vote













                                  The brief answer to this is 'because the centre of mass of a bicycle is high up', and especially it is high up compared to a car, where often the front wheels can slip.



                                  To see how this works, consider a vastly oversimplified model of a bike: assume that the structure of the bike is light compared to its rider, and represent its rider as a point mass (see below for why this works even when that's not true). And we're going to be interested in the moment when the rear wheel lifts, so we can completely ignore the rear wheel, and concentrate just on the front wheel, and specifically on the point at which the front wheel touches the road. So the system looks something like this:



                                  enter image description here



                                  So, here, $c$ is the point at which the front tyre is touching the road, $m$ is the rider, and the horizontal & vertical distances between $c$ and $m$ are $l$ & $h$ respectively. And the bike is decelerating at $a$. And I've drawn the forces exerted by the front wheel on the road at $c$ (remember the back wheel is, by assumption, just lifting, so can be ignored: it's not exerting any forces on anything).



                                  For now assume that the coefficient of friction between the front wheel and the road is sufficiently high that the wheel does not slip and lets work out the point at which the bike just begins to tumble over the front wheel: this will tell us the maximum possible value of $a$, however sticky the front wheel is.



                                  It's pretty easy to see that the force on $m$ has two components: a vertical component which is $-mg$, where $g$ is acceleration due to gravity, and a horizontal component which is $ma$, where $a$ is the horizontal acceleration. And the bike will tumble when this vector points above the front wheel. Well, just by drawing the appropriate components you can see that this is true when



                                  $$frac{ma}{mg} gt frac{l}{h}$$



                                  or in other words, for the bike not to tumble



                                  $$a le frac{lg}{h}$$



                                  or



                                  $$a_text{max} = frac{lg}{h} tag{1}$$



                                  You can convince yourself this is right: a very tall bike ($h gg l$) will tumble really easily, and a completely flat bike ($h ll l$) will almost never tumble. And a bike in very low gravity will tumble more easily than one in high gravity. So (1) tells us how big $a$ can be, however sticky the front wheel is.



                                  Now consider the coefficient of friction at the front wheel. The coefficient of friction, $mu$ is defined as the force with which the wheel is trying to slide along the road and the force which it is being pressed down onto the road, at the point where the wheel just slips. So it's obvious that,



                                  $$mu = frac{ma_text{slip}}{mg} = frac{a_text{slip}}{g}$$



                                  where $a_text{slip}$ is the point at which the wheel slips. In other words



                                  $$a_text{slip} = mu g tag{2}$$



                                  And now we can use (1) & (2) to give us the answer we're looking for: the bike will tumble before it drifts if $a_text{slip} gt a_text{max}$, in other words if



                                  $$mu gt frac{l}{h}$$



                                  And you can now see the problem here: bikes are rather short and rather tall, so $l/h$ tends to be rather small, which means that the bike will tumble over the front wheel with a lower critical value of $mu$. And modern tyres on dry roads have values of $mu$ which can be fairly close to $1$ (I think $0.8$ to $0.9$ is plausible), while $l/h$ is generally significantly less than $1$.



                                  This is why bikes tumble before they drift.





                                  This approximation can be used even if the bike (or other vehicle) isn't light compared to the rider: you just need to work out where the centre of mass of the vehicle is and use that. For vehicles with suspension (some bikes have this of course, and even for bikes that don't the forks deflect under braking) you have to take into account the change in geometry under braking as well.






                                  share|cite|improve this answer

























                                    up vote
                                    2
                                    down vote










                                    up vote
                                    2
                                    down vote









                                    The brief answer to this is 'because the centre of mass of a bicycle is high up', and especially it is high up compared to a car, where often the front wheels can slip.



                                    To see how this works, consider a vastly oversimplified model of a bike: assume that the structure of the bike is light compared to its rider, and represent its rider as a point mass (see below for why this works even when that's not true). And we're going to be interested in the moment when the rear wheel lifts, so we can completely ignore the rear wheel, and concentrate just on the front wheel, and specifically on the point at which the front wheel touches the road. So the system looks something like this:



                                    enter image description here



                                    So, here, $c$ is the point at which the front tyre is touching the road, $m$ is the rider, and the horizontal & vertical distances between $c$ and $m$ are $l$ & $h$ respectively. And the bike is decelerating at $a$. And I've drawn the forces exerted by the front wheel on the road at $c$ (remember the back wheel is, by assumption, just lifting, so can be ignored: it's not exerting any forces on anything).



                                    For now assume that the coefficient of friction between the front wheel and the road is sufficiently high that the wheel does not slip and lets work out the point at which the bike just begins to tumble over the front wheel: this will tell us the maximum possible value of $a$, however sticky the front wheel is.



                                    It's pretty easy to see that the force on $m$ has two components: a vertical component which is $-mg$, where $g$ is acceleration due to gravity, and a horizontal component which is $ma$, where $a$ is the horizontal acceleration. And the bike will tumble when this vector points above the front wheel. Well, just by drawing the appropriate components you can see that this is true when



                                    $$frac{ma}{mg} gt frac{l}{h}$$



                                    or in other words, for the bike not to tumble



                                    $$a le frac{lg}{h}$$



                                    or



                                    $$a_text{max} = frac{lg}{h} tag{1}$$



                                    You can convince yourself this is right: a very tall bike ($h gg l$) will tumble really easily, and a completely flat bike ($h ll l$) will almost never tumble. And a bike in very low gravity will tumble more easily than one in high gravity. So (1) tells us how big $a$ can be, however sticky the front wheel is.



                                    Now consider the coefficient of friction at the front wheel. The coefficient of friction, $mu$ is defined as the force with which the wheel is trying to slide along the road and the force which it is being pressed down onto the road, at the point where the wheel just slips. So it's obvious that,



                                    $$mu = frac{ma_text{slip}}{mg} = frac{a_text{slip}}{g}$$



                                    where $a_text{slip}$ is the point at which the wheel slips. In other words



                                    $$a_text{slip} = mu g tag{2}$$



                                    And now we can use (1) & (2) to give us the answer we're looking for: the bike will tumble before it drifts if $a_text{slip} gt a_text{max}$, in other words if



                                    $$mu gt frac{l}{h}$$



                                    And you can now see the problem here: bikes are rather short and rather tall, so $l/h$ tends to be rather small, which means that the bike will tumble over the front wheel with a lower critical value of $mu$. And modern tyres on dry roads have values of $mu$ which can be fairly close to $1$ (I think $0.8$ to $0.9$ is plausible), while $l/h$ is generally significantly less than $1$.



                                    This is why bikes tumble before they drift.





                                    This approximation can be used even if the bike (or other vehicle) isn't light compared to the rider: you just need to work out where the centre of mass of the vehicle is and use that. For vehicles with suspension (some bikes have this of course, and even for bikes that don't the forks deflect under braking) you have to take into account the change in geometry under braking as well.






                                    share|cite|improve this answer














                                    The brief answer to this is 'because the centre of mass of a bicycle is high up', and especially it is high up compared to a car, where often the front wheels can slip.



                                    To see how this works, consider a vastly oversimplified model of a bike: assume that the structure of the bike is light compared to its rider, and represent its rider as a point mass (see below for why this works even when that's not true). And we're going to be interested in the moment when the rear wheel lifts, so we can completely ignore the rear wheel, and concentrate just on the front wheel, and specifically on the point at which the front wheel touches the road. So the system looks something like this:



                                    enter image description here



                                    So, here, $c$ is the point at which the front tyre is touching the road, $m$ is the rider, and the horizontal & vertical distances between $c$ and $m$ are $l$ & $h$ respectively. And the bike is decelerating at $a$. And I've drawn the forces exerted by the front wheel on the road at $c$ (remember the back wheel is, by assumption, just lifting, so can be ignored: it's not exerting any forces on anything).



                                    For now assume that the coefficient of friction between the front wheel and the road is sufficiently high that the wheel does not slip and lets work out the point at which the bike just begins to tumble over the front wheel: this will tell us the maximum possible value of $a$, however sticky the front wheel is.



                                    It's pretty easy to see that the force on $m$ has two components: a vertical component which is $-mg$, where $g$ is acceleration due to gravity, and a horizontal component which is $ma$, where $a$ is the horizontal acceleration. And the bike will tumble when this vector points above the front wheel. Well, just by drawing the appropriate components you can see that this is true when



                                    $$frac{ma}{mg} gt frac{l}{h}$$



                                    or in other words, for the bike not to tumble



                                    $$a le frac{lg}{h}$$



                                    or



                                    $$a_text{max} = frac{lg}{h} tag{1}$$



                                    You can convince yourself this is right: a very tall bike ($h gg l$) will tumble really easily, and a completely flat bike ($h ll l$) will almost never tumble. And a bike in very low gravity will tumble more easily than one in high gravity. So (1) tells us how big $a$ can be, however sticky the front wheel is.



                                    Now consider the coefficient of friction at the front wheel. The coefficient of friction, $mu$ is defined as the force with which the wheel is trying to slide along the road and the force which it is being pressed down onto the road, at the point where the wheel just slips. So it's obvious that,



                                    $$mu = frac{ma_text{slip}}{mg} = frac{a_text{slip}}{g}$$



                                    where $a_text{slip}$ is the point at which the wheel slips. In other words



                                    $$a_text{slip} = mu g tag{2}$$



                                    And now we can use (1) & (2) to give us the answer we're looking for: the bike will tumble before it drifts if $a_text{slip} gt a_text{max}$, in other words if



                                    $$mu gt frac{l}{h}$$



                                    And you can now see the problem here: bikes are rather short and rather tall, so $l/h$ tends to be rather small, which means that the bike will tumble over the front wheel with a lower critical value of $mu$. And modern tyres on dry roads have values of $mu$ which can be fairly close to $1$ (I think $0.8$ to $0.9$ is plausible), while $l/h$ is generally significantly less than $1$.



                                    This is why bikes tumble before they drift.





                                    This approximation can be used even if the bike (or other vehicle) isn't light compared to the rider: you just need to work out where the centre of mass of the vehicle is and use that. For vehicles with suspension (some bikes have this of course, and even for bikes that don't the forks deflect under braking) you have to take into account the change in geometry under braking as well.







                                    share|cite|improve this answer














                                    share|cite|improve this answer



                                    share|cite|improve this answer








                                    edited 14 hours ago

























                                    answered 15 hours ago









                                    tfb

                                    14.4k42848




                                    14.4k42848






















                                        up vote
                                        0
                                        down vote













                                        I can't answer why but I can tell you that it is a property of multi-wheeled vehicles. Take a toy car and block the rear wheels so they don't move and push it on a smooth surface and you will see the locked rear wheels will always swap to the front. This is why a "bootleg turn" is possible by using the rear emergency brake to lock them down and the car will swap directions without changing the original vector by much. This is why the front brakes on a car does 90% of the braking and why the brakes on the front wheel of a motorcycle can help maintain control far more than the rear.






                                        share|cite|improve this answer








                                        New contributor




                                        Kathmandu Gilman is a new contributor to this site. Take care in asking for clarification, commenting, and answering.
                                        Check out our Code of Conduct.






















                                          up vote
                                          0
                                          down vote













                                          I can't answer why but I can tell you that it is a property of multi-wheeled vehicles. Take a toy car and block the rear wheels so they don't move and push it on a smooth surface and you will see the locked rear wheels will always swap to the front. This is why a "bootleg turn" is possible by using the rear emergency brake to lock them down and the car will swap directions without changing the original vector by much. This is why the front brakes on a car does 90% of the braking and why the brakes on the front wheel of a motorcycle can help maintain control far more than the rear.






                                          share|cite|improve this answer








                                          New contributor




                                          Kathmandu Gilman is a new contributor to this site. Take care in asking for clarification, commenting, and answering.
                                          Check out our Code of Conduct.




















                                            up vote
                                            0
                                            down vote










                                            up vote
                                            0
                                            down vote









                                            I can't answer why but I can tell you that it is a property of multi-wheeled vehicles. Take a toy car and block the rear wheels so they don't move and push it on a smooth surface and you will see the locked rear wheels will always swap to the front. This is why a "bootleg turn" is possible by using the rear emergency brake to lock them down and the car will swap directions without changing the original vector by much. This is why the front brakes on a car does 90% of the braking and why the brakes on the front wheel of a motorcycle can help maintain control far more than the rear.






                                            share|cite|improve this answer








                                            New contributor




                                            Kathmandu Gilman is a new contributor to this site. Take care in asking for clarification, commenting, and answering.
                                            Check out our Code of Conduct.









                                            I can't answer why but I can tell you that it is a property of multi-wheeled vehicles. Take a toy car and block the rear wheels so they don't move and push it on a smooth surface and you will see the locked rear wheels will always swap to the front. This is why a "bootleg turn" is possible by using the rear emergency brake to lock them down and the car will swap directions without changing the original vector by much. This is why the front brakes on a car does 90% of the braking and why the brakes on the front wheel of a motorcycle can help maintain control far more than the rear.







                                            share|cite|improve this answer








                                            New contributor




                                            Kathmandu Gilman is a new contributor to this site. Take care in asking for clarification, commenting, and answering.
                                            Check out our Code of Conduct.









                                            share|cite|improve this answer



                                            share|cite|improve this answer






                                            New contributor




                                            Kathmandu Gilman is a new contributor to this site. Take care in asking for clarification, commenting, and answering.
                                            Check out our Code of Conduct.









                                            answered yesterday









                                            Kathmandu Gilman

                                            61




                                            61




                                            New contributor




                                            Kathmandu Gilman is a new contributor to this site. Take care in asking for clarification, commenting, and answering.
                                            Check out our Code of Conduct.





                                            New contributor





                                            Kathmandu Gilman is a new contributor to this site. Take care in asking for clarification, commenting, and answering.
                                            Check out our Code of Conduct.






                                            Kathmandu Gilman is a new contributor to this site. Take care in asking for clarification, commenting, and answering.
                                            Check out our Code of Conduct.






















                                                up vote
                                                0
                                                down vote













                                                In the front wheel you have a much stronger vertical component when you use the breaks because of the momentum caused by inertia. With the distribution of forces and weight of a normal person, the back wheel will tend to lift adding more friction I'm the front wheel and stopping you harder and harder. To drift you need to have inertia in the back wheel but when you stop like this all the energy is released stopping you and lifting your back wheel



                                                When you use the break of the back wheel only, the momentum caused by inertia tends to lift the back wheel, such as in the case above. That causes friction to be lower I'm the back wheel and thus energy is not released suddenly, allowing you to drift.



                                                I believe that if you used a wheel with bad grip in the front wheel you should be able to drift






                                                share|cite|improve this answer








                                                New contributor




                                                F. S. is a new contributor to this site. Take care in asking for clarification, commenting, and answering.
                                                Check out our Code of Conduct.






















                                                  up vote
                                                  0
                                                  down vote













                                                  In the front wheel you have a much stronger vertical component when you use the breaks because of the momentum caused by inertia. With the distribution of forces and weight of a normal person, the back wheel will tend to lift adding more friction I'm the front wheel and stopping you harder and harder. To drift you need to have inertia in the back wheel but when you stop like this all the energy is released stopping you and lifting your back wheel



                                                  When you use the break of the back wheel only, the momentum caused by inertia tends to lift the back wheel, such as in the case above. That causes friction to be lower I'm the back wheel and thus energy is not released suddenly, allowing you to drift.



                                                  I believe that if you used a wheel with bad grip in the front wheel you should be able to drift






                                                  share|cite|improve this answer








                                                  New contributor




                                                  F. S. is a new contributor to this site. Take care in asking for clarification, commenting, and answering.
                                                  Check out our Code of Conduct.




















                                                    up vote
                                                    0
                                                    down vote










                                                    up vote
                                                    0
                                                    down vote









                                                    In the front wheel you have a much stronger vertical component when you use the breaks because of the momentum caused by inertia. With the distribution of forces and weight of a normal person, the back wheel will tend to lift adding more friction I'm the front wheel and stopping you harder and harder. To drift you need to have inertia in the back wheel but when you stop like this all the energy is released stopping you and lifting your back wheel



                                                    When you use the break of the back wheel only, the momentum caused by inertia tends to lift the back wheel, such as in the case above. That causes friction to be lower I'm the back wheel and thus energy is not released suddenly, allowing you to drift.



                                                    I believe that if you used a wheel with bad grip in the front wheel you should be able to drift






                                                    share|cite|improve this answer








                                                    New contributor




                                                    F. S. is a new contributor to this site. Take care in asking for clarification, commenting, and answering.
                                                    Check out our Code of Conduct.









                                                    In the front wheel you have a much stronger vertical component when you use the breaks because of the momentum caused by inertia. With the distribution of forces and weight of a normal person, the back wheel will tend to lift adding more friction I'm the front wheel and stopping you harder and harder. To drift you need to have inertia in the back wheel but when you stop like this all the energy is released stopping you and lifting your back wheel



                                                    When you use the break of the back wheel only, the momentum caused by inertia tends to lift the back wheel, such as in the case above. That causes friction to be lower I'm the back wheel and thus energy is not released suddenly, allowing you to drift.



                                                    I believe that if you used a wheel with bad grip in the front wheel you should be able to drift







                                                    share|cite|improve this answer








                                                    New contributor




                                                    F. S. is a new contributor to this site. Take care in asking for clarification, commenting, and answering.
                                                    Check out our Code of Conduct.









                                                    share|cite|improve this answer



                                                    share|cite|improve this answer






                                                    New contributor




                                                    F. S. is a new contributor to this site. Take care in asking for clarification, commenting, and answering.
                                                    Check out our Code of Conduct.









                                                    answered 18 hours ago









                                                    F. S.

                                                    1




                                                    1




                                                    New contributor




                                                    F. S. is a new contributor to this site. Take care in asking for clarification, commenting, and answering.
                                                    Check out our Code of Conduct.





                                                    New contributor





                                                    F. S. is a new contributor to this site. Take care in asking for clarification, commenting, and answering.
                                                    Check out our Code of Conduct.






                                                    F. S. is a new contributor to this site. Take care in asking for clarification, commenting, and answering.
                                                    Check out our Code of Conduct.






















                                                        up vote
                                                        0
                                                        down vote













                                                        It's because the force of friction is approximately propertional to two things:




                                                        • The "coefficient of friction" between the two surfaces (i.e. how sticky they are)

                                                        • The "weight" or normal force (perpendicular force) between the two surfaces.


                                                        The latter means, for example, that if you double the weight on a sled then you more or less double the friction between the sled and the ground.



                                                        If you move all (or most of) your weight off the back wheel and onto the front wheel of a bike, then:




                                                        • The force due to friction on the front wheel more or less doubles (i.e. you'd need twice as much sideways or backward force to make it skid).

                                                        • The force due to friction on the back wheel goes towards zero (i.e. it skids more easily as it begins t lift off the ground)






                                                        share|cite|improve this answer

























                                                          up vote
                                                          0
                                                          down vote













                                                          It's because the force of friction is approximately propertional to two things:




                                                          • The "coefficient of friction" between the two surfaces (i.e. how sticky they are)

                                                          • The "weight" or normal force (perpendicular force) between the two surfaces.


                                                          The latter means, for example, that if you double the weight on a sled then you more or less double the friction between the sled and the ground.



                                                          If you move all (or most of) your weight off the back wheel and onto the front wheel of a bike, then:




                                                          • The force due to friction on the front wheel more or less doubles (i.e. you'd need twice as much sideways or backward force to make it skid).

                                                          • The force due to friction on the back wheel goes towards zero (i.e. it skids more easily as it begins t lift off the ground)






                                                          share|cite|improve this answer























                                                            up vote
                                                            0
                                                            down vote










                                                            up vote
                                                            0
                                                            down vote









                                                            It's because the force of friction is approximately propertional to two things:




                                                            • The "coefficient of friction" between the two surfaces (i.e. how sticky they are)

                                                            • The "weight" or normal force (perpendicular force) between the two surfaces.


                                                            The latter means, for example, that if you double the weight on a sled then you more or less double the friction between the sled and the ground.



                                                            If you move all (or most of) your weight off the back wheel and onto the front wheel of a bike, then:




                                                            • The force due to friction on the front wheel more or less doubles (i.e. you'd need twice as much sideways or backward force to make it skid).

                                                            • The force due to friction on the back wheel goes towards zero (i.e. it skids more easily as it begins t lift off the ground)






                                                            share|cite|improve this answer












                                                            It's because the force of friction is approximately propertional to two things:




                                                            • The "coefficient of friction" between the two surfaces (i.e. how sticky they are)

                                                            • The "weight" or normal force (perpendicular force) between the two surfaces.


                                                            The latter means, for example, that if you double the weight on a sled then you more or less double the friction between the sled and the ground.



                                                            If you move all (or most of) your weight off the back wheel and onto the front wheel of a bike, then:




                                                            • The force due to friction on the front wheel more or less doubles (i.e. you'd need twice as much sideways or backward force to make it skid).

                                                            • The force due to friction on the back wheel goes towards zero (i.e. it skids more easily as it begins t lift off the ground)







                                                            share|cite|improve this answer












                                                            share|cite|improve this answer



                                                            share|cite|improve this answer










                                                            answered 17 hours ago









                                                            ChrisW

                                                            33618




                                                            33618






















                                                                up vote
                                                                -3
                                                                down vote













                                                                I think its because you can turn the handle bars into the best angle to maintain your center of gravity near the body.



                                                                While the back wheel cant turn so looses grip because the center of gravity goes outside your body during a drift






                                                                share|cite|improve this answer

























                                                                  up vote
                                                                  -3
                                                                  down vote













                                                                  I think its because you can turn the handle bars into the best angle to maintain your center of gravity near the body.



                                                                  While the back wheel cant turn so looses grip because the center of gravity goes outside your body during a drift






                                                                  share|cite|improve this answer























                                                                    up vote
                                                                    -3
                                                                    down vote










                                                                    up vote
                                                                    -3
                                                                    down vote









                                                                    I think its because you can turn the handle bars into the best angle to maintain your center of gravity near the body.



                                                                    While the back wheel cant turn so looses grip because the center of gravity goes outside your body during a drift






                                                                    share|cite|improve this answer












                                                                    I think its because you can turn the handle bars into the best angle to maintain your center of gravity near the body.



                                                                    While the back wheel cant turn so looses grip because the center of gravity goes outside your body during a drift







                                                                    share|cite|improve this answer












                                                                    share|cite|improve this answer



                                                                    share|cite|improve this answer










                                                                    answered yesterday









                                                                    eromod

                                                                    896




                                                                    896

















                                                                        protected by ACuriousMind 11 hours ago



                                                                        Thank you for your interest in this question.
                                                                        Because it has attracted low-quality or spam answers that had to be removed, posting an answer now requires 10 reputation on this site (the association bonus does not count).



                                                                        Would you like to answer one of these unanswered questions instead?



                                                                        Popular posts from this blog

                                                                        QoS: MAC-Priority for clients behind a repeater

                                                                        Ивакино (Тотемский район)

                                                                        Can't locate Autom4te/ChannelDefs.pm in @INC (when it definitely is there)